Solution manual engineering economic analysis 9th edition ch07 rate of return analysis

25 183 0
Solution manual   engineering economic analysis 9th edition ch07 rate of return analysis

Đang tải... (xem toàn văn)

Tài liệu hạn chế xem trước, để xem đầy đủ mời bạn chọn Tải xuống

Thông tin tài liệu

Chapter 7: Rate of Return Analysis 7-1 $12 $10 $20 $30 $40 $50 $60 $125 = $10 (P/A, i%, 6) + $10 (P/G, i%, 6) at 12%, $10 (4.111) + $10 (8.930) = $130.4 at 15%, $10 (3.784) + $10 (7.937) = $117.2 i* = 12% + (3%) ((130.4 – 125).(130.4-117.2)) = 13.23% 7-2 $80 $80 $80 $200 $200 $80 $80 $80 $200 The easiest solution is to solve one cycle of the repeating diagram: $80 $80 $80 = $20 $120 = $80 (F/P, i%, 1) $120 = $80 (1 + i) (1 + i) = $120/$80 = 1.50 $12 i* = 0.50 = 50% Alternative Solution: EUAB = EUAC $80 = [$200 (P/F, i%, 2) + $200 (P/F, i%, 4) + $200 (P/F, i%, 6)] (A/P, i%, 6) Try i = 50% $80 = [$200 (0.4444) + $200 (0.1975) + $200 (0.0878)] (0.5481) Therefore i* = 50% 7-3 $5 $10 $15 $20 $25 $42.55 $42.55 = $5 (P/A, i%, 5) + $5 (P/G, i%, 5) Try i = 15%, $5 (3.352) + $5 (5.775) = $45.64 > $42.55 Try i = 20%, $5 (2.991) + $5 (4.906) = $39.49 < $42.55 Rate of Return = 15% + (5%) [($45.64 - $42.55)/($45.64 - $39.49)] = 17.51% Exact Answer: 17.38% 7-4 For infinite series: A = Pi EUAC= EUAB $3,810 (i) = $250 + $250 (F/P, i%, 1) (A/F, i%, 2)* Try i = 10% $250 + $250 (1.10) (0.4762) = $381 $3,810 (0.10) = $381 i = 10% *Alternate Equations: $3,810 (i) = $250 + $250 (P/F, i%, 1) (A/P, i%, 2) $3,810 (j) = $500 - $250 (A/G, i%, 2) = $79.99 7-5 P’ A = $1,000 ………… Yr n = 10 $41 n=∞ $5,000 At Year 0, PW of Cost = PW of Benefits $412 + $5,000 (P/F, i%, 10) = ($1000/i) (P/F, i%, 10) Try i = 15% $412 + $5,000 (0.2472) $1,648 = ($1,000/0.15) (0.2472) = $1,648 ROR = 15% 7-6 The algebraic sum of the cash flows equals zero Therefore, the rate of return is 0% 7-7 A = $300 $1,000 Try i = 5% $1,000 =(?) Try i = 6% $1,000 =(?) $300 (3.546) (0.9524) =(?) $1,013.16 $300 (3.465) (0.9434) =(?) $980.66 Performing Linear Interpolation: i* = 5% + (1%) (($1,013.6 - $1,000)/($1,013.6 - $980.66)) = 5.4% 7-8 $400 = [$200 (P/A, i%, 4) - $50 (P/G, i%, 4)] (P/F, i%, 1) Try i = 7% [$200 (3.387) - $50 (4.795)] (0.9346) = 409.03 Try i = 8% [$200 (3.312) - $50 (4.650)] (0.9259) = $398.08 i* = 7% + (1%) [($409.03 - $400)/($409.03 - $398.04)] = 7.82% 7-9 $100 (P/A, i%, 10) = $27 (P/A, i%, 10) = 3.704 Performing Linear Interpolation: (P/A, i%, 10) i 4.192 20% 3.571 25% Rate of Return = 20% + (5%) [(4.192 – 3.704)/(4.912 – 3.571)] = 23.9% 7-10 Year Cash Flow -$500 -$100 +$300 +$300 +$400 +$500 $500 + $100 (P/F, i%, 1)= $300 (P/A, i%, 2) (P/F, i%, 1) + $400 (P/F, i%, 4) + $500 (P/F, i%, 5) Try i = 30% $500 + $100 (0.7692) = $576.92 $300 (1.361) (0.7692) + $400 (0.6501) + $500 (0.2693) = $588.75 ∆ = 11.83 Try i = 35% $500 + $100 (0.7407) = $574.07 $300 (1.289) (0.7407) + $400 (0.3011) + $500 (0.2230) = $518.37 ∆ = 55.70 Rate of Return = 30% + (5%) [11.83/55.70) = 31.06% Exact Answer: 30.81% 7-11 Year 10 Cash Flow -$223 -$223 -$223 -$223 -$223 -$223 +$1,000 +$1,000 +$1,000 +$1,000 +$1,000 The rate of return may be computed by any conventional means On closer inspection one observes that each $223 increases to $1,000 in five years $223 = $1,000 (P/F, i%, 5) (P/F, i%, 5) = $223/$1,000 = 0.2230 From interest tables, Rate of Return = 35% 7-12 Year Cash Flow -$640 40 +$100 +$200 +$300 +$300 $640 = $100 (P/G, i%, 4) + $300 (P/F, i%, 5) Try i = 9% $100 (4.511) + $300 (0.6499) = $646.07 > $640 Try i = 10% $100 (4.378) + $300 (0.6209) = $624.07 < $640 Rate of Return = 9% + (1%) [(%646.07 - $640)/($646.07 - $624.07)] = 9.28% 7-13 Since the rate of return exceeds 60%, the tables are useless F = P (1 + i)n $4,500 = $500 (1 + i)4 (1 + i)4 = $4,500/$500 =0 (1 + i) = 91/4 = 1.732 i* = 0.732 = 73.2% 7-14 $3,000 = $119.67 (P/A, i%, 30) (P/A, i%, 30) = $3,000/$119.67 = 25.069 Performing Linear Interpolation: (P/A, i%, 30) 25.808 24.889 i i 1% 1.25% = 1% + (0.25%)((25.808-25.069)/(25.808-24.889)) = 1.201% (a) Nominal Interest Rate = 1.201 x 12 = 14.41% (b) Effective Interest Rate = (1 + 0.01201)12 – = 0.154 = 15.4% 7-15 $3,000 A = $325 ………… n = 36 $12,375 $9,375 = $325 (P/A, i%, 36) (P/A, i%, 36) = $9,375/$325 = 28.846 From compound interest tables, i = 1.25% Nominal Interest Rate Effective Interest Rate = 1.25 x 12 = 15% = (1 + 0.0125)12 – = 16.08% 7-16 1991 – 1626 = 365 years = n F = P (1 + i)n 12 x 109 = 24 (1 + i)365 (1 + i)365 = 12 x 100/24 = 5.00 x 108 This may be immediately solved on most hand calculators: i* = 5.64% Solution based on compound interest tables: (F/P, i%, 365) = 5.00 x 108 = (F/P, i%, 100) (F/P, i%, 100) (F/P, i%, 100) (F/P, i%, 65) Try i = 6% (F/P, 6%, 365) = (339.3)3 (44.14) = 17.24 X 108 (i too high) Try i = 5% (F/P, 5%, 365) = (131.5)3 (23.84) = 0.542 X 108 (i too low) Performing linear interpolation: i* = 5% + (1%) [((5 – 0.54) (108))/((17.24 – 0.54) (108))] = 5% + 4.46/16.70 = 5.27% The linear interpolation is inaccurate 7-17 $1,000 A = $40 n = 10 $92 PW of Cost = PW of Benefits $925 = $40 (P/A, i%, 10) + $1,000 (P/F, i%, 10) Try i = 5% $925 = $40 (7.722) + $1,000 (0.6139) = $922.78 (i too high) Try i = 4.5% $925 = $40 (7.913) + $1,000 (0.6439) = $960.42 (i too low) i* ≈ 4.97% 7-18 $1,000 A = $40 …… n = 40 semiannual periods $715 PW of Benefits – PW of Costs = $20 (P/A, i%, 40) + $1,000 (P/F, i%, 40) - $715 = Try i = 3% $20 (23.115) + $1,000 (0.3066) - $715 = $53.90 i too low Try i = 3.5% $20 (21.355) + $1,000 (0.2526) - $715 = -$35.30 i too high Performing linear interpolation: i* = 3% + (0.5%) [53.90/(53.90 – (-35.30))] = 3.30% Nominal i* = 6.60% 7-19 $12,000 $6,000 $3,000 n = 10 n = 10 $28,000 PW of Cost = PW of Benefits n = 20 $28,000 = $3,000 (P/A, i%, 10) + $6,000 (P/A, i%, 10) (P/F, i%, 10) + $12,000 (P/A, i%, 20) (P/F, i%, 20) Try i = 12% $3,000 (5.650) + $6,000 (5.650) (0.3220) + $12,000 (7.469) (0.1037) = $37,160 > $28,000 Try i = 15% $3,000 (5.019) + $6,000 (5.019) (0.2472) + $12,000 (6.259) (0.0611) = $27,090 < $28,000 Performing Linear Interpolation: i* = 15% - (3%) [($28,000 - $27,090)/($37,160 - $27,090)] = 15% - (3%) (910/10,070) = 14.73% 7-20 $15,000 A = $80 $9,000 PW of Benefits – PW of Cost = $0 $15,000 (P/F, i%, 4) - $9,000 - $80 (P/A, i%, 4) = $0 Try i = 12% $15,000 (0.6355) - $9,000 - $80 (3.037) = +$289.54 Try i = 15% $15,000 (0.5718) - $9,000 - $80 (2.855) = -$651.40 Performing Linear Interpolation: i* = 12% + (3%) [289.54/(289.54 + 651.40)] = 12.92% 7-21 The problem requires an estimate for n- the expected life of the infant Seventy or seventyfive years might be the range of reasonable estimates Here we will use 71 years The purchase of a $200 life subscription avoids the series of beginning-of-year payments of $12.90 Based on 71 beginning-of-year payments, A = $12.90 ………… n = 70 $200 $200 - $12.90 (P/A, i%, 70) 6% < i* < 8%, = $12,90 (P/A, i%, 70) = $187.10/$12.90 = 14.50 By Calculator: i* = 6.83% 7-22 $1,000 A = $30 ……… n = 2(2001 – 1998) + = 27 $875 PW of Benefits – PW of Cost = $0 $30 (P/A, i%, 27) + $1,000 (P/F, i%, 27) - $875 = $0 Try i = ½% $30 (17.285) + $1,000 (0.3950) - $875 = $38.55 >$0 Try i = 4% $30 (16.330) + $1,000 (0.3468) - $875 = -$38.30 < $0 i* = 3.75% Nominal rate of return = (3.75%) = 7.5% 7-23 A = $110 ……… n = 24 $3,500 - $1,200 = $2,300 $2,300 = $110 (P/A, i%, 24) (P/A, i%, 24) = $2,300/$110 = 20.91 From tables: 1% < i < 1.25% On Financial Calculator: i = 1.13% per month Effective interest rate = (1 + 0.0113)12 – = 0.144 = 14.4% 7-24 A = $100 ……… n = 36 $3,168 PW of Cost = PW of Benefits $100 (P/A, i%, 36)= $3,168 (P/A, i%, 36) = $3,168/$100 = 31.68 Performing Linear Interpolation: (P/A, i%, 36) I 32.871 ẵ% 21.447 ắ% i* = (1/2%) + (1/4%) [(32.87 – 31.68)/(32.87 – 31.45)] = 0.71% Nominal Interest Rate = 12 (0.71%) = 8.5% 7-25 This is a thought-provoking problem for which there is no single answer Two possible solutions are provided below A Assuming the MS degree is obtained by attending graduate school at night while continuing with a full-time job: Cost: $1,500 per year for years Benefit: $3,000 per year for 10 years MS Degree A = $3,000 n = 10 $1,500 $1,500 Computation as of award of MS degree: $1,500 (F/A, i%, 2) = $3,000 (P/A, i%, 10) i* > 60 B Assuming the MS degree is obtained by one of year of full-time study Cost: Difference between working & going to school Whether working or at school there are living expenses The cost of the degree might be $24,000 Benefit: $3,000 per year for 10 years $24,000 = $3,000 (P/A, i%, 10) i* = 4.3% 7-26 $35 A = $12.64 ……… n = 12 $175 ($175 - $35) = $12.64 (P/A, i%, 12) (P/A, i%, 12) = $140/$12.64 = 11.08 i = ¼% Nominal interest rate = 12 (1 ¼%) = 15% 7-27 The rate of return exceeds 60% so the interest tables are not useful F $25,000 (1 + i) i* = P (1 + i)n = $5,000 (1 + i)3 = ($25,000/$5,000)1/3 = 1.71 = 0.71 Rate of Return = 71% 7-28 This is an unusual problem with an extremely high rate of return Available interest tables obviously are useless One may write: PW of Cost = PW of Benefits $0.5 = $3.5 (1 + i)-1 + $0.9 (1 + i)-2 + $3.9 (1 + i)-3 + $8.6 (1 + i)-4 + … For high interest rates only the first few terms of the series are significant: Try i = 650% PW of Benefits = $3.5/(1 + 6.5) + $0.9/(1 + 6.5)2 + $3.9/(1 + 6.5)3 + $8.6/(1 + 6.5)4 + … = 0.467 + 0.016 + 0.009 + 0.003 = 0.495 Try i = 640% PW of Benefits = $3.5/(1 + 6.4) + $0.9/(1 + 6.4)2 + $3.9/(1 + 6.4)3 + $8.6/(1 + 6.4)4 + … = 0.473 + 0.016 + 0.010 + 0.003 = 0.502 i* ≈ 642% (Calculator Solution: i = 642.9%) 7-29 g = 10% A1 = $1,100 n = 20 i=? P = $20,000 The payment schedule represents a geometric gradient There are two possibilities: i ≠ g and i = g Try the easier i = g computation first: P = A1n (1 + i)-1 where g = i = 0.10 $20,000 = $1,100 (20) (1.10)-1 = $20,000 Rate of Return i* = g = 10% 7-30 (a) Using Equation (4-39): F = Pern $4,000 = $2,000er(9) = er(9) 9r = ln = 0.693 r = 7.70% (b) Equation (4-34) ieff = er – = e0.077 – = 0.0800 = 8.00% 7-31 (a) When n = ∞, i = A/P = $3,180/$100,000 = 3.18% (b) (A/P, i%, 100) = $3180/$100,000 From interest tables, i* = 3% = 0.318 (c) (A/P, i%, 50) = $3,180/$100,000 From interest tables, i* = 2% = 0.318 (d) The saving in water truck expense is just a small part of the benefits of the pipeline Convenience, improved quality of life, increased value of the dwellings, etc., all are benefits Thus, the pipeline appears justified 7-32 F = $2,242 A = $50 n=4 P = $1,845 Set PW of Cost = PW of Benefits $1,845 = $50 (P/A, i%, 4) + $2,242 (P/F, i%, 4) Try i = 7% 450 (3.387) + $2,242 (0.7629) = $1,879 > $1,845 Try i = 8% 450 (3.312) + $2,242 (0.7350) = $1,813 < $1,845 Rate of Return = 7% + (1%) [($1,879 - $1,845)/($1,879 - $1,813)] = 7.52% for months Nominal annual rate of return = (7.52%) = 15.0% Equivalent annual rate of return = (1 + 0.0752)2 – = 15.6% 7-33 (a) F = $5 P = $1 n=5 F = P (1 + i)n $5 = $1 (1 + i)5 (1 + i) = 50.20 = 1.38 i* = 38% (b) For a 100% annual rate of return F = $1 (1 + 1.0)5 = $32, not $5! Note that the prices Diagonal charges not necessarily reflect what anyone will pay a collector for his/her stamps 7-34 $800 $400 $6,000 $9,000 Year 1- 5- Cash Flow -$9,000 +$800 +$400 +$6,000 PW of Cost = PW of Benefits $9,000 = $400 (P/A, i%, 8) + $400 (P/A, i%, 4) + $6,000 (P/F, i%, 9) Try i = 3% $400 (7.020) + $400 (3.717) + $6,000 (0.7664) = $8,893 < $9,000 Try i = ½% $400 (7.170) + $400 (3.762) + $6,000 (0.8007) = $9,177 > $9,000 Rate of Return = ½% + (1/2%) [($9,177 - $9,000)/($9,177 - $8,893)] = 2.81% 7-35 Year Cash Flow -$1,000 +$1,094.60 +$1,094.60 $1,000 = $1,094 [(P/F, i%, 6) + (P/F, i%, 9)] Try i = 20% $1,094 [(0.5787) + (0.3349)] = $1,000 Rate of Return = 20% 7-36 $65,000 $5,000 $240,000 $240,000 = $65,000 (P/A, i%, 13) - $5,000 (P/G, i%, 13) Try i = 15% $65,000 (5.583) -$5,000 (23.135) = $247,220 > $240,000 Try i = 18% $65,000 (4.910) -$5,000 (18.877) = $224,465 < $240,000 Rate of Return = 15% + 3% [($247,220 - $240,000)/($247,220 - $224,765)] = 15.96% 7-37 3,000 = 30 (P/A, i*, 120) (P/A, i*, 120) = 3,000/30 = 100 Performing Linear Interpolation: (P/A, i%, 120) 103.563 100 90.074 i* I ẳ% i* ẵ% = 0.0025 + 0.0025 [(103.562 – 100)/(103.562 – 90.074)] = 0.00316 per month Nominal Annual Rate = 12 (0.00316) = 0.03792 = 3.79% 7-38 (a) Total Annual Revenues = $500 (12 months) (4 apt.) = $24,000 Annual Revenues – Expenses = $24,000 - $8,000 = $16,000 To find Internal Rate of Return the Net Present Worth must be $0 NPW = $16,000 (P/A, i*, 5) + $160,000 (P/F, i*, 5) - $140,000 At i = 12%, At i = 15%, IRR NPW = $8,464 NPW = -$6,816 = 12% + (3%) [$8,464/($8,464 + $6,816)] = 13.7% (b) At 13.7% the apartment building is more attractive than the other options 7-39 NPW = -$300,000 + $20,000 (P/F, i*, 10) + ($67,000 - $3,000) (P/A, i*, 10) - $600 (P/G, i*, 10) Try i = 10% NPW= -$300,000 + $20,000 (0.3855) + ($64,000) (6.145) - $600 (22.891) = $87,255 > $0 The interest rate is too low Try i = 18% NPW = -$300,000 + $20,000 (0.1911) + ($64,000) (4.494) - $600 (14.352) = -$17,173 < $0 The interest rate is too high Try i = 15% NPW = -$300,000 + $20,000 (0.2472) + ($64,000) (5.019) - $600 (16.979) = $9,130 > $0 Thus, the rate of return (IRR) is between 15% and 18% By linear interpolation: i* = 15% + (3%) [$9,130/($9,130 - $17,173)] = 16.0% 7-40 (a) First payment = $2, Final payment = 132 x $2 = $264 Average Payment = ($264 + $2)/2 = $133 Total Amount = 132 payments x $133 average pmt = $17,556 Alternate Solution: The payments are same as sum-of-years digits form SUM = (n/2) (n + 1) = (132/2) (133) = 8,778 Total Amount = $2 (8778) = $17,556 (b) The bank will lend the present worth of the gradient series Loan (P) = $2 (P/G, 1%, 133) Note: n- = 132, so n = 133 By interpolation, (P/G, 1%, 133) = 3,334.11 + (13/120) (6,878.6 – 3,334.1) = 3,718.1 Loan (P) = $2 (3,718.1) = $7,436.20 7-41 Year 10 11 12 … 33 34 35 36 Case (incl Deposit) -$39,264.00 +$599.00 +$599.00 +$599.00 +$599.00 +$599.00 +$599.00 +$599.00 +$599.00 +$599.00 +$599.00 +$599.00 +$599.00 +$599.00 +$599.00 +$599.00 +$599.00 +$27,854.00 -$625.00 = +$27,229.00 IRR Nominal IRR Effective IRR = 0.86% = 10.32% = 10.83% 7-42 MARR = 5% P = $30,000 n = 35 years Alternative 1: Withdraw $15,000 today and lose $15,000 Alternative 2: Wait, leave your fund in the system until retirement Equivalency seeks to determine what future amount is equal to $15,000 now F = P (1 + i)n = $30,000 (1.05)35 = $30,000 (5.516015) = $165,480.46 Therefore: $15,000 = $165,480.46 (1 + i)-35 $15,000 (1 + i)35 = $165,480.46 (1 + i) = [(165,480.46/$15,000)]1/35 i = 1.071 – = 7.1002% > 5% Unless $15,000 can be invested with a return higher than 7.1%, it is better to wait for 35 years for the retirement fund $15,000 now is only equivalent to $165,480.46 35 years from now if the interest rate now is 7.1% instead of the quoted 5% 7-43 $2,000 = $91.05 (P/A, i*, 30) (P/A, i*, 30) = $2,000/$91.05 = 21.966 (P/A, i%, 30) i 22.396 20.930 2½ imo = 2% + (1/2%) [(22.396 – 21.966)/(22.396 – 20.930)] = 2.15% per month Nominal ROR received by finance company = 12 (2.15%) = 25.8% 7-44 $3,000 = $118.90 (P/A, i*, 36) (P/A, i*, 36) = $3,000/$118.90 = 26.771 (P/A, i%, 36) i 27.661 ẵ% 26.543 ắ% imo = ½% + ¼% [(27.661 – 26.771)/(27.661 – 26.543)] = 1.699% per month Nominal Annual ROR 7-45 (a) = 12 (1.699%) = 20.4% ... $1,813 < $1,845 Rate of Return = 7% + (1%) [($1,879 - $1,845)/($1,879 - $1,813)] = 7.52% for months Nominal annual rate of return = (7.52%) = 15.0% Equivalent annual rate of return = (1 + 0.0752)2... 10% $100 (4.378) + $300 (0.6209) = $624.07 < $640 Rate of Return = 9% + (1%) [(%646.07 - $640)/($646.07 - $624.07)] = 9.28% 7-13 Since the rate of return exceeds 60%, the tables are useless F =... rate = 12 (1 ¼%) = 15% 7-27 The rate of return exceeds 60% so the interest tables are not useful F $25,000 (1 + i) i* = P (1 + i)n = $5,000 (1 + i)3 = ($25,000/$5,000)1/3 = 1.71 = 0.71 Rate of

Ngày đăng: 13/09/2018, 10:28

Từ khóa liên quan

Mục lục

  • Chapter 7: Rate of Return Analysis

    • 7-1

    • 7-2

    • 7-3

    • 7-4

    • 7-5

    • 7-6

    • 7-7

    • 7-8

    • 7-9

    • 7-10

    • 7-11

    • 7-12

    • 7-13

    • 7-14

    • 7-15

    • 7-16

    • 7-17

    • 7-18

    • 7-19

    • 7-20

Tài liệu cùng người dùng

Tài liệu liên quan